Sei sulla pagina 1di 64

1

SIMPLE STRESS NORMAL D=119.35 mm answer For steel cable:


Pst=σstAst
Problem104 Problem 105
A hollow steel tube with an inside diameter A homogeneous 800 kg bar AB is supported 3924=120Ast
of 100 mm must carry a tensile load of 400 at either end by a cable as shown in Fig. P-
Ast=32.7mm2 answer
kN. Determine the outside diameter of the 105. Calculate the smallest area of each
tube if the stress is limited to 120 MN/m2. cable if the stress is not to exceed 90 MPa Problem 106
in bronze and 120 MPa in steel.
Solution 104 The homogeneous bar shown in Fig. P-106
is supported by a smooth pin at C and a
P=σA
cable that runs from A to B around the
where: smooth peg at D. Find the stress in the cable
P=400kN=400000N if its diameter is 0.6 inch and the bar weighs
6000 lb.
σ=120MPa

A=14πD2−14π(1002)

A=14π(D2−10000) Solution 105

By symmetry:
Pbr=Pst=12(7848)

Pbr=3924 N

Solution 106

ΣMC=0

5T+10[(3/√34)T]=5(6000)
For bronze cable: T=2957.13lb
Thus, Pbr=σbrAbr
400000=120[14π(D2−10000)]
3924=90Abr
400000=30πD2−300000π
Abr=43.6mm2 answer
2
D2=400000+300000π30πD
2

T=σA For aluminum: For bronze:


σalAal=Pal σbrAbr=2P
2957.13=σ[1/4π(0.62)]
σal(0.5)=12 100(200)=2P
σ=10458.72 psi answer
σal=24 ksi answer P=10000N
Problem 107
A rod is composed of an aluminum section
rigidly attached between steel and bronze For bronze:
sections, as shown in Fig. P-107. Axial loads σbrAbr=Pbr
are applied at the positions indicated. If P =
σbr(0.5)=9
3000 lb and the cross sectional area of the
rod is 0.5 in2, determine the stress in each σbr=18 ksi answer
section.

Problem 108 For aluminum:


An aluminum rod is rigidly attached σalAal=P
between a steel rod and a bronze rod as
90(400)=P
shown in Fig. P-108. Axial loads are applied
at the positions indicated. Find the P=36000N
For steel: maximum value of P that will not exceed a
σstAst=Pst stress in steel of 140 MPa, in aluminum of
For Steel:
90 MPa, or in bronze of 100 MPa.
σst(0.5)=12 σstAst=5P

σst=24 ksi answer P=14000N

For safe value of P, use the smallest above.


Thus,

P=10 000 N=10 kN answer


3

Problem 109 For wire AC: For wood:


Determine the largest weight W that can be TAC/sin60∘=W/sin80∘ Pw=σwAw
supported by two wires shown in Fig. P-109.
TAC=0.8794W Pw=1800[1/4π(82)]
The stress in either wire is not to exceed 30
ksi. The cross-sectional areas of wires AB TAC=σACAAC Pw=90477.9lb
and AC are 0.4 in2 and 0.5 in2, respectively.
0.8794W=30(0.5)

W=17.1kips From FBD of Wood:

For safe load W,

W=17.1 kips answer

Free body diagram of Joint A


P=Pw=90477.9lb
Problem 110
A 12-inches square steel bearing plate lies
between an 8-inches diameter wooden For concrete:
post and a concrete footing as shown in Fig. Pc=σcAc
P-110. Determine the maximum value of
the load P if the stress in wood is limited to Pc=650(122)
1800 psi and that in concrete to 650 psi. Pc=93600lb
For wire AB: By sine law (from the force
polygon):
From FBD of Concrete:
TAB/sin40∘=W/sin80∘

TAB=0.6527W
P=Pc=93600lb
σABAAB=0.6527W
For safe load P,
30(0.4)=0.6527W
P=90478lb answer
W=18.4kips
4

Problem 111 Stresses: (Stress = Force/Area)


For the truss shown in Fig. P-111, calculate σCE=[16(2/3)]/1.8=9.26ksi (Tension)
the stresses in members CE, DE, and DF. The
answer
cross-sectional area of each member is 1.8
in2. Indicate tension (T) or compression (C). σDE=40/1.8=22.22ksi (Tension)

By symmetry answer
BD=DF=33(1/3)k(Compression) σDF=[33(1/3)]/1.8=18.52ksi (Compression)

answer
ΣFV=0 Problem 112
DE=3/5BD+3/5DF Determine the cross-sectional areas of
members AG, BC, and CE for the truss
ΣMA=0
DE=3/5[33(1/3)]+3/5[33(1/3)] shown in Fig. P-112. The stresses are not to
24RF=16(30) exceed 20 ksi in tension and 14 ksi in
DE=40k(Tension)
compression. A reduced stress in
RF=20k
compression is specified to reduce the
At joint F: danger of buckling.
At joint E:

ΣFV=0
ΣFV=0
3/5DF=20
3/5CE+30=40
DF=33(1/3)k(Compression)
CE=16(2/3)k(Tension)

At joint D:
5

ΣFV=0 For member AG (At joint A): ΣMF=0

RAV=40+25=65k 6(2/√13)BC=12(20)

ΣAV=0 BC=72.11k Compression

18RD=8(25)+4(40)
BC=σcompressionABC
RD=20k ΣFV=0
72.11=14ABC
ΣFH=0 (3/√13)AB=65
ABC=5.15in2 answer
RAH=RD=20k AB=78.12 k

ΣFH=0 For member CE (At joint D):


AG+20=(2/√13)AB

AG=23.33kTension

AG=σtensionAAG

23.33=20AAG

AAG=1.17in2 answer

ΣFH=0
For member BC (At section through MN)
Check: (2/√13)CD=20
ΣMD=0 CD=36.06k
12RAV=18(RAH)+4(25)+8(40)

12(65)=18(20)+4(25)+8(40) ΣFV=0

780 ft⋅kip=780 ft⋅kip (OK!) DE=(3/√13)CD=(3/√13)(36.06)=30k


6

At joint E: CF=275.77kN Compression

CF=σCFA

275.77(1000)=σCF(1600)

σCF=172.357MPa
(Compression) answer
For member BD: (See FBD 01)
ΣFV=0 ΣMC=0 For member BC: (See FBD 02)
(3/√13)EF=30 3(4/5BD)=3(60)
EF=36.06k BD=75kN Tension
ΣFH=0

CE=(2/√13)EF=(2/√13)(36.06)=20k

Compression

CF=σcompressionACE ΣMD=0
20=14ACE 4BC=7(60)
ACE=1.43in2 answer BD=σBDA BC=105kN Compression
Problem 113 75(1000)=σBD(1600)
Find the stresses in members BC, BD, and CF BC=σBCA
for the truss shown in Fig. P-113. Indicate σBD=46.875MPa (Tension) answer
the tension or compression. The cross 105(1000)=σBC(1600)
sectional area of each member is 1600 For member CF: (See FBD 01) σBC=65.625MPa
mm2. ΣMD=0 (Compression) answer
4(1/√2CF)=4(90)+7(60)
7

Problem 114 Based on cable at C:


The homogeneous bar ABCD shown in Fig. T2=σCAC
ΣFV=0
P-114 is supported by a cable that runs
0.1256W=100(300)
from A to B around the smooth peg at E, a TABsin30∘+TAB+TC+RDcos50∘=W
vertical cable at C, and a smooth inclined W=238853.50N
surface at D. Determine the mass of the TABsin30∘+TAB+TC+(1.1305TAB)cos50∘=W
heaviest bar that can be supported if the 2.2267TAB+TC=W
stress in each cable is limited to 100 MPa. Sfave value of W
The area of the cable AB is 250 mm2 and TC=W−2.2267TAB W=63661.83N
that of the cable at C is 300 mm2. W=mg
ΣMD=0
63661.83=m(9.81)
6(TAB sin30∘)+4T AB+2TC=3W
m=6489.5kg
7TAB+2(W−2.2267TAB)=3W
m=6.49Mg answer
2.5466TAB=W

TAB=0.3927W

Solution 114 TC=W−2.2267TAB

ΣFH=0 TC=W−2.2267(0.3927W)

TABcos30∘=RDsin50∘ TC=0.1256W

RD=1.1305TAB
Based on cable AB:

TAB=σABAAB

0.3927W=100(250)

W=63661.83N
8

SIMPLE STRESS SHEAR limited to 50 ksi. (a) Compute the maximum Based on compression of puncher:
thickness of plate in which a hole 2.5 inches P=σA
Problem 115
in diameter can be punched. (b) If the plate
What force is required to punch a 20-mm- P=50(1/4πd2)
is 0.25 inch thick, determine the diameter
diameter hole in a plate that is 25 mm
of the smallest hole that can be punched. P=12.5πd2 → Equivalent shear force for
thick? The shear strength is 350 MN/m2.
plate

Solution 115
Based on shearing of plate:
The resisting area is the shaded area along
V=τA → V=P
the perimeter and the shear force VV is
equal to the punching force PP. 12.5πd2=40[πd(0.25)]
Solution 116
d=0.8in answer
(a) Maximum thickness of plate:

Based on puncher strength:


P=σA Problem 117
Find the smallest diameter bolt that can be
P=50[1/4π(2.52)] used in the clevis shown in Fig. 1-11b if P =
400 kN. The shearing strength of the bolt is
P=78.125πkips → Equivalent shear
300 MPa.
force of the plate
V=τA
Based on shear strength of plate:
P=350[π(20)(25)]
V=τA → V=P
P=549778.7N
78.125π=40[π(2.5t)]
P=549.8kN answer
t=0.781inch answer Solution 117
Problem 116
As in Fig. 1-11c, a hole is to be punched out The bolt is subject to double shear.
of a plate having a shearing strength of 40 V=τA
ksi. The compressive stress in the punch is (b) Diameter of smallest hole:
9

400(1000)=300[2(1/4πd2)] F=73.33kN From the FBD:


ΣMC=0
d=29.13mm answer
V=τA 0.25RBV=0.25(40sin35∘)+0.2(40cos35∘)
Problem 118
A 200-mm-diameter pulley is prevented Where: RBV=49.156kN
from rotating relative to 60-mm-diameter V=F=73.33kN
shaft by a 70-mm-long key, as shown in Fig.
A=70b
P-118. If a torque T = 2.2 kN·m is applied to
the shaft, determine the width b if the τ=60MPa
allowable shearing stress in the key is 60
MPa. Thus,
73.33(1000)=60(70b)

b=17.46mm answer ΣFH=0

RBH=40cos35∘

Problem 119 RBH=32.766kN


Compute the shearing stress in the pin at B
for the member supported as shown in Fig. RB=√(RBH2+RBV2)
P-119. The pin diameter is 20 mm.
Solution 118
RB=√(32.7662+49.1562)

RB=59.076kN → shear force of pin at B

VB=τBA → double shear

59.076(1000)=τB{2[1/4π(202)]}

τB=94.02 MPa answer

T=0.03F Solution 119

2.2=0.03F
10

4RBH+4RBV=2WAB 2RBH−RBV=671 → Equation (2)

Problem 120 4RBH+4RBV=2(1132) 3RBH=1237


The members of the structure in Fig. P-120
RBH+RBV=566 → Equation (1) RBH=412.33lb
weigh 200 lb/ft. Determine the smallest
diameter pin that can be used at A if the For member BC:
shearing stress is limited to 5000 psi.
Assume single shear. From equation (1):
412.33+RBV=566

RBV=153.67lb

From the FBD of member AB


ΣFH=0

RAH=RBH=412.33lb

Solution 120
Length, LBC=√(32+62)=6.71ft ΣFV=0
For member AB:
Weight, WBC=6.71(200)=WBC=1342lb RAV+RBV=WAB
ΣMC=0 RAV+153.67=1132
6RBH=1.5WBC+3RBV RAV=978.33lb
6RBH−3RBV=1.5(1342)
RA=√R2AH+R2AV
2RBH−RBV=671 → Equation (2)
RA=√(412.332+978.332)
Add equations (1) and (2) RA=1061.67lb → shear force of pin at A
Length, LAB=√(42+42)−−−−−−=5.66ft

Weight, WAB=5.66(200)=1132lb RBH+RBV=566 → Equation (1) V=τA


ΣMA=0 2RBH−RBV=671 1061.67=5000(1/4πd2)
11

d=0.520in answer BH=Tcos10∘ RB=17.48P

Problem 121 P=RB/17.48 → Equation (2)


Referring to Fig. P-121, compute the From Equation (1),
maximum force P that can be applied by the T=3Psin10∘
machine operator, if the shearing stress in Based on tension of rod (equation 1):
the pin at B and the axial stress in the P=(1/3)Tsin10∘
control rod at C are limited to 4000 psi and Thus, P=1/3[5000×1/4π(0.5)2]sin10∘
5000 psi, respectively. The diameters are BH=(3P/sin10∘)cos10∘
0.25 inch for the pin, and 0.5 inch for the P=56.83lb
control rod. Assume single shear for the pin BH=3cot10∘P
at B.
Based on shear of rivet (equation 2):
ΣFV=0 P={4000[1/4π(0.25)2]}/17.48

BV=Tsin10∘+P P=11.23lb

Safe load P,
Again from Equation (1),
Solution 121 Tsin10∘=3P P=11.23 lb answer

ΣMB=0

6P=2Tsin10∘ Thus, Problem 122


BV=3P+P Two blocks of wood, width w and thickness
3P=Tsin10∘ → Equation (1) t, are glued together along the joint inclined
BV=4P at the angle θ as shown in Fig. P-122. Using
the free-body diagram concept in Fig. 1-4a,
show that the shearing stress on the glued
RB2=BH2+BV2 joint is τ = P sin 2θ / 2A, where A is the
cross-sectional area.
RB2=(3cot10∘P)2+(4P)2
ΣFH=0 R2B=305.47P2
12

V=τAshear Solution 123

Pcosθ=τ(Acscθ) Based on maximum


compressive stress:
τ=(Psinθcosθ)/A
Normal force:
τ=[P(2sinθcosθ)]/2A
N=Pcos20∘
τ=Psin2θ/2A (okay!)

Normal area:
Problem 123 AN=50(100sec20∘)
A rectangular piece of wood, 50 mm by 100
AN=5320.89mm2
mm in cross section, is used as a
compression block shown in Fig. P-123.
Determine the axial force P that can be
N=σAN
safely applied to the block if the
compressive stress in wood is limited to 20 Pcos20∘=20(5320.89)
Solution 122 MN/m2 and the shearing stress parallel to
P=113247N
the grain is limited to 5MN/m2. The grain
Shear area, makes an angle of 20° with the horizontal, P=133.25kN
Ashear=t(wcscθ) as shown. (Hint: Use the results in Problem
122.)
Ashear=twcscθ
Based on maximum shearing stress:
Ashear=Acscθ
Shear force:
V=Psin20∘
Shear force,
V=Pcosθ Shear area:
AV=AN

AV=5320.89mm2
13

V=τAV

Psin20∘=5(5320.89)

P=77786N

P=77.79kN

For safe compressive force use

P=77.79 kN answer
14

SIMPLE STRESS BEARING t=7.85mm answer Based on shearing of rivets:


P=τA
Problem 125
Part (b): Largest average tensile stress in P=14[4((1/4)π)(34)2]
In Fig. 1-12, assume that a 20-mm-diameter
the plate:
rivet joins the plates that are each 110 mm P=24.74kips
P=σA
wide. The allowable stresses are 120 MPa
for bearing in the plate material and 60 6000π=σ[7.85(110−20)]
Based on bearing of plates:
MPa for shearing of rivet. Determine (a) the
σ=26.67MPa answer P=σbAb
minimum thickness of each plate; and (b)
the largest average tensile stress in the P=18[4(3/4)(7/8)]
plates.
Problem 126 P=47.25kips
The lap joint shown in Fig. P-126 is fastened
by four ¾-in.-diameter rivets. Calculate the
Safe load P,
maximum safe load P that can be applied if
the shearing stress in the rivets is limited to P=24.74kips answer
14 ksi and the bearing stress in the plates is
limited to 18 ksi. Assume the applied load is Problem 127
Solution 125 uniformly distributed among the four In the clevis shown in Fig. 1-11b, find the
rivets. minimum bolt diameter and the minimum
Part (a): thickness of each yoke that will support a
From shearing of rivet: load P = 14 kips without exceeding a
P=τArivets shearing stress of 12 ksi and a bearing
P=60[1/4π(202)] stress of 20 ksi.

P=6000πN

From bearing of plate material:


P=σbAb

6000π=120(20t)
Solution 126
15

Problem 128 Designation Web thickness


A W18 × 86 beam is riveted to a W24 × 117
girder by a connection similar to that in Fig.
W18 × 86 0.480 in
1-13. The diameter of the rivets is 7/8 in.,
and the angles are each L4 × 3-1/2 × 3/8 in..
For each rivet, assume that the allowable W24 × 117 0.550 in
stresses are τ = 15 ksi and σb = 32 ksi. Find
Shearing strength of rivets:
the allowable load on the connection.
There are 8 single-shear rivets in the girder
and 4 double-shear (equivalent to 8 single-
shear) in the beam, thus, the shear strength
Solution 127 of rivets in girder and beam are equal.
For shearing of rivets (double shear) V=τA=15[1/4π(7/8)2(8)]
P=τA V=72.16 kips
14=12[2(1/4πd2)]

d=0.8618in → diameter of Bearing strength on the girder:


bolt answer The thickness of girder W24 × 117 is 0.550
Given:
Shape of beam = W18 × 86 inch while that of the angle clip L4 × 3-1/2 ×
For bearing of yoke: Shape of girder = W24 × 117 3/8 is 3/8 or 0.375 inch, thus, the critical in
P=σbAb Shape of angles = L4 × 3-1/2 × 3/8 bearing is the clip.
Diameter of rivets = 7/8 inch P=σbAb=32[7/8(0.375)(8)]
14=20[2(0.8618t)] Allowable shear stress = 15 ksi
P=84 kips
t=0.4061in → thickness of Allowable bearing stress = 32 ksi
yoke answer
Required:
Bearing strength on the beam:
Allowable load on the connection
The thickness of beam W18 × 86 is 0.480
Relevant data from the table (Appendix B inch while that of the clip angle is 2 × 0.375
of textbook): Properties of Wide-Flange = 0.75 inch (clip angles are on both sides of
Sections (W shapes): U.S. Customary Units the beam), thus, the critical in bearing is the
beam.
P=σbAb=32[7/8(0.480)(4)]
16

P=53.76 kips Inside diameter of washer = 9/8 inch


Tensile stress in the nut = 18 ksi
The allowable load on the connection is
Bearing stress = 800 psi
P=53.76kips answer
Required:
Shearing stress in the head of the bolt
Shearing stress in threads of the bolt
Outside diameter of the washer

Problem 129 Tensile force on the bolt:


A 7/8-in.-diameter bolt, having a diameter P=σA=18[1/4π(7/8)2]
at the root of the threads of 0.731 in., is P=10.82kips
used to fasten two timbers together as Problem 130
shown in Fig. P-129. The nut is tightened to Shearing stress in the head of the bolt:
Figure P-130 shows a roof truss and the
cause a tensile stress of 18 ksi in the bolt. τ=P/A=10.82/[π(7/8)(1/2)]
detail of the riveted connection at joint B.
Compute the shearing stress in the head of Using allowable stresses of τ = 70 MPa
τ=7.872 ksi answer
the bolt and in the threads. Also, determine and σb= 140 MPa, how many 19-mm-
the outside diameter of the washers if their diameter rivets are required to fasten
inside diameter is 9/8 in. and the bearing Shearing stress in the threads: member BC to the gusset plate? Member
stress is limited to 800 psi. τ=P/A=10.82/[π(0.731)(5/8)] BE? What is the largest average tensile or
τ=7.538 ksi answer compressive stress in BC and BE?

Outside diameter of washer:


P=σbAb

10.82(1000)=800{1/4π[d2−(9/8)2]}

d=4.3inch answer
Given:
Diameter of bolt = 7/8 inch
Diameter at the root of the thread (bolt) =
0.731 inch
17

Based on shearing of rivets:

BE=τA Where A = area of 1 rivet × number


of rivets, n

80000=70[1/4π(192)n]

n=4.03 say 5 rivets

Based on bearing of member:


For Member BC:
BE=σbAb Where Ab = rivet diameter ×
Based on shearing of rivets: thickness of BE × n rivets
BC=τA Where A = area of 1 rivet × 80000=140[19(13)n]
number of rivets, n
At Joint C: n=2.3 say 3 rivets
ΣFV=0 96000=70[1/4π(192)n]

BC=96kN (Tension) n=4.8 say 5 rivets Use 5 rivets for member BE. answer

Consider the section through member BD, Based on bearing of member:


Relevant data from the table (Appendix B
BE, and CE: BC=σbAb Where Ab = rivet diameter × of textbook): Properties of Equal Angle
ΣMA=0 thickness of BC × n rivets Sections: SI Units
8(3/5BE)=4(96) 96000=140[19(6)n]
Designation Area
BE=80kN (Compression) n=6.02 say 7 rivets
L75 × 75 × 6 864 mm2
Use 7 rivets for member BC. answer
L75 × 75 × 13 1780 mm2

For member BE:


18

Based on bearing of member: Use 4 rivets for member BE answer


P=σbAb
Tensile stress of member BC
(L75 × 75 × 6): 96000=140[22(6)n] Compressive stress:
σ=P/A=80(1000)/1780
n=5.2 say 6 rivets
σ=44.94MPa answer
σ=P/A=[96(1000)]/[864−19(6)]
Use 6 rivets for member BC. answer THIN-WALLED PRESSURE VESSEL
σ=128Mpa answer
Problem 133
Tensile stress: A cylindrical steel pressure vessel 400 mm
Compressive stress of member BE (L75 ×
σ=P/A=96(1000)/[864−22(6)] in diameter with a wall thickness of 20 mm,
75 × 13):
is subjected to an internal pressure of 4.5
σ=131.15MPa answer
σ=P/A=[80(1000)]/1780 MN/m2. (a) Calculate the tangential and
longitudinal stresses in the steel. (b) To
σ=44.94Mpa answer
what value may the internal pressure be
For member BE: increased if the stress in the steel is limited
P=80kN (Compression) to 120 MN/m2? (c) If the internal pressure
Problem 131 were increased until the vessel burst,
Repeat Problem 130 if the rivet diameter is sketch the type of fracture that would
22 mm and all other data remain Based on shearing of rivets: occur.
unchanged. P=τA
For member BC: 80000=70[1/4π(222)n] Part (a)
P=96kN (Tension) n=3.01 say 4 rivets Tangential stress (longitudinal section):

Based on shearing of rivets: Based on bearing of member:


P=τA P=σbAb
96000=70[1/4π(222)n] 80000=140[22(13)n]
n=3.6 say 4 rivets n=1.998 say 2 rivets F=2T

pDL=2(σttL)
19

σt=pD/2t=4.5(400)/2(20) The bursting force will cause a stress in the Total internal pressure:
longitudinal section that is twice to that of P=p(1/4πD2)
σt=45MPa answer
the transverse section. Thus, fracture is
expected as shown.
Resisting wall:
Longitudinal Stress (transverse section):
F=P

σA=p(1/4πD2)

σ(πDt)=p(1/4πD2)

σ=pD/4t

8000=[p(4×12)]/4(516)

p=208.33psi answer
Problem 134
F=P The wall thickness of a 4-ft-diameter
spherical tank is 5/16 inch. Calculate the Problem 135
1/4πD2p=σl(πDt)
allowable internal pressure if the stress is Calculate the minimum wall thickness for a
σl=pD/4t=4.5(400)/4(20) limited to 8000 psi. cylindrical vessel that is to carry a gas at a
pressure of 1400 psi. The diameter of the
σl=22.5MPa answer
Solution 134 vessel is 2 ft, and the stress is limited to 12
Part (b) ksi.

From (a), σt=pD/2t and σl=pD/4t

thus, σt=2σl, this shows that tangential The critical stress is the tangential stress
stress is the critical. σt=pD/2t
σt=pD2t
12000=1400(2×12)/2t
120=p(400)/2(20)
t=1.4 in answer
p=12MPa answer
20

Problem 136 p=5.33MPa be filled if the circumferential stress is


A cylindrical pressure vessel is fabricated limited to 6000 psi. The specific weight
from steel plating that has a thickness of 20 ofwater is 62.4 lb/ft3.
Based on longitudinal stress:
mm. The diameter of the pressure vessel is
450 mm and its length is 2.0 m. Determine
Solution 137
the maximum internal pressure that can be
applied if the longitudinal stress is limited
to 140 MPa, and the circumferential stress
is limited to 60 MPa.

Solution 136

Based on circumferential stress


(tangential): ΣFH=0

F=P σt=6000psi=6000lb/in2(12in/ft)2

p(1/4πD2)=σl(πD) σt=864000lb/ft2

σl=pD4t
Assuming pressure distribution to be
140=p(450)/4(20)
uniform:
p=24.89MPa p=γh=62.4h

ΣFV=0 Use p=5.33MPa F=pA=62.4h(Dh)

F=2T answer F=62.4(22)h2

p(DL)=2(σtLt)

σt=pD2t Problem 137


A water tank, 22 ft in diameter, is made
60=p(450)2(20) from steel plates that are 1/2 in. thick. Find
the maximum height to which the tank may
21

F=1372.8h2

T=σtAt=864000(th) Problem 138


The strength of longitudinal joint in Fig. 1-
T=864000(1/2×1/12)h
17 is 33 kips/ft, whereas for the girth is 16
T=36000h kips/ft. Calculate the maximum diameter of
the cylinder tank if the internal pressure is
150 psi.
ΣF=0

F=2T

1372.8h2=2(36000h)
Total hydrostatic force, F:
h=52.45ft answer FF = volume of pressure diagram
COMMENT F=1/2(γh2)D=1/2(62.4h2)(22)
Given a free surface of water, the actual
pressure distribution on the vessel is not F=686.4h2
uniform. It varies linearly from zero at the
free surface to γh at the bottom (see figure ΣMA=0
below). Using this actual pressure Solution 138
distribution, the total hydrostatic pressure
For longitudinal joint (tangential stress):
is reduced by 50%. This reduction of force
will take our design into critical situation;
giving us a maximum height of 200% more
than the h above.
2T(1/2h)−F(1/3h)=0
Based on actual pressure distribution: T=1/3F

σt(ht)=1/3(686.4h2)

h=3σtt/686.4=3(864000)(12×112)/686.4

h=157.34ft
22

Consider 1 ft length D=35.56in. answer CF=2γAv2/g


F=2T

pD=2σtt 2T=CF
Problem 139
σt=pD/2t Find the limiting peripheral velocity of a 2γA=2γAv2/g
rotating steel ring if the allowable stress is
33000/t=21600D/2t σ=γv2/g
20 ksi and steel weighs 490 lb/ft3. At what
D=3.06ft=36.67in. revolutions per minute (rpm) will the stress
reach 30 ksi if the mean radius is 10 in.? From the given data:
σ=20ksi=(20000lb/in2)(12in/ft)a2
For girth joint (longitudinal stress):
Solution 139 σ=2880000lb/ft2

γ=490lb/ft3

2880000=490v2/32.2

v=435.04ft/sec answer

When

σ=30ksi,and R=10in
Centrifugal Force, CF:
σ=γv2/g
F=P CF=Mω2x¯
30000(122)=490v2/32.2
p(1/4πD2)=σl(πDt) where:
M=(W/g) (γV/g)=γπRA/q v=532.81ft/sec
σl=pD/4t
ω=v/R
16000/t=21600D/4t
ω=v/R=532.81/(10/12)
x¯=2R/π
D=2.96ft=35.56in.
ω=639.37rad/sec
Thus,
CF=(γπRA/q)(v/R)2(2R/π) ω=(639.37rad/sec)( 1rev/2πrad)
Use the smaller diameter,
(60sec/1min)
23

ω=6,105.54rpm answer CF=2ρAR2ω2

2T=CF
Problem 140
At what angular velocity will the stress of 2σA=2ρAR2ω2
the rotating steel ring equal 150 MPa if its
σ=ρR2ω2
mean radius is 220 mm? The density of
steel 7.85 Mg/m3.
From the given (Note: 1 N = 1 kg·m/sec2):
σ=150MPa Solution 141
Solution 140
σ=150000000kg⋅m/sec2⋅m2 Longitudinal Stress:
σ=150000000kg/m⋅sec2 F=pA=125[1.5(2)+1/4π(1.5)2](122)
ρ=7.85Mg/m3=7850kg/m3 F=85808.62lbs
R=220mm=0.22m

Therefore,
CF=Mω x¯ 2 150000000=7850(0.22)2ω2

ω=628.33rad/sec answer

Problem 141
Where: The tank shown in Fig. P-141 is fabricated
M=ρV=ρAπR from 1/8-in steel plate. Calculate the
P=F
maximum longitudinal and circumferential
x=2R/π stress caused by an internal pressure of 125 σl[2(2×12)(1/8)+π(1.5×12)(1/8)]=85808.62
psi.
σl=6566.02psi
Thus, σl=6.57ksi answer
CF=ρAπRω2(2R/π)
24

Circumferential Stress: AXIAL DEFORMATION Plot the stress-strain diagram and


determine the following mechanical
F=pA=125[(2×12)L+2(0.75×12)L] Problem 203
properties: (a) proportional limits; (b)
The following data were recorded during
F=5250L lbs modulus of elasticity; (c) yield point; (d)
the tensile test of a 14-mm-diameter mild
ultimate strength; and (e) rupture strength.
steel rod. The gage length was 50 mm.

Solution 203
Elongation Load Elongation
Load (N) Area, A = 0.25π(142) = 49π mm2
(mm) (N) (mm)
Length, L = 50 mm
Strain = Elongation/Length
0 0 46 200 1.25
Stress = Load/Area

6 310 0.010 52 400 2.50


Load Elongation Strain Stress
12 600 0.020 58 500 4.50 (N) (mm) (mm/mm) (MPa)

2T=F 18 800 0.030 68 000 7.50 0 0 0 0


2[σt(1/8)L]=5250L
25 100 0.040 59 000 12.5 6 310 0.010 0.0002 40.99
σt=21000psi

σt=21ksi answer 31 300 0.050 67 800 15.5 12 600 0.020 0.0004 81.85

37 900 0.060 65 000 20.0 18 800 0.030 0.0006 122.13

40 100 0.163 65 500 Fracture 25 100 0.040 0.0008 163.05

31 300 0.050 0.001 203.33


41 600 0.433

37 900 0.060 0.0012 246.20


25

40 100 0.163 0.0033 260.49 initial diameter of the test specimen was
0.505 in. and the gage length was 2.0 in.
41 600 0.433 0.0087 270.24
Load Elongatio Load Elongatio
46 200 1.250 0.025 300.12 (lb) n (in.) (lb) n (in.)

52 400 2.500 0.05 340.40 14 00


0 0 0.020
0
58 500 4.500 0.09 380.02
14 40
2 310 0.00220 0.025
68 000 7.500 0.15 441.74 0

59 000 12.500 0.25 383.27 14 50


From stress-strain diagram: 4 640 0.00440 0.060
0
67 800 15.500 0.31 440.44 1. Proportional Limit = 246.20 MPa
14 60
2. Modulus of Elasticity 6 950 0.00660 0.080
65 000 20.000 0.4 422.25 0
E = slope of stress-strain diagram
within proportional limi
61 500 Failure 399.51 14 80
E = 246.20/0.0012 = 205 166.67 9 290 0.00880 0.100
0
MPa = 205.2 GPa

3. Yield Point = 270.24 MPa 11 60 14 60


0.0110 0.120
0 0
4. Ultimate Strength = 441.74 MPa

5. Rupture Strength = 399.51 MPa 12 60 13 60


0.0150 Fracture
0 0

Problem 204
The following data were obtained during a
tension test of an aluminum alloy. The
26

Plot the stress-strain diagram and 0.004 46 381.3


determine the following mechanical 9 290 0.0088
4 2
properties: (a) proportional limit; (b)
modulus of elasticity; (c) yield point; (d)
11 60 0.005 57 914.2
yield strength at 0.2% offset; (e) ultimate 0.011
0 5 4
strength; and (f) rupture strength.

12 60 0.007 62 906.8
0.015
Solution 204 0 5 5
Area, A = 0.25π(0.5052) = 0.0638π in2
Length, L = 2 in 14 00 69 896.4
0.02 0.01
Strain = Elongation/Length 0 9
Stress = Load/Area
14 40 0.012 71 893.5
0.025
0 5 4 From stress-strain diagram:
Load Elongatio Strain Stress
(lb) n (in.) (in/in) (psi) 1. Proportional Limit = 57,914.24 psi
14 50 72 392.8
0.06 0.03
0 0 2. Modulus of Elasticity:
0 0 0 0 E = 57914.24/0.0055 =
14 60 72 892.0 10,529,861.82 psi
0.08 0.04
0.001 11 532.9 0 6 E = 10,529.86 ksi
2 310 0.0022
1 2
3. Yield Point = 69,896.49 psi
14 80 73 890.5
0.1 0.05
0.002 23 165.7 0 8 4. Yield Strength at 0.2% Offset:
4 640 0.0044
2 0 Strain of Elastic Limit = ε at PL +
14 60 72 892.0 0.002
0.12 0.06 Strain of Elastic Limit = 0.0055 +
0.003 34 698.6 0 6
6 950 0.0066 0.002
3 2
Strain of Elastic Limit = 0.0075
13 60 67 899.4
Fracture in/in
0 5
27

The offset line will pass through Q (See σ1 = 1804.84 psi δ=ρg/E∫0Lydy=ρg/E[y2/2]0L
figure below):
δ=ρg/2E[L2−02]
Yield Strength at 0.2% Offset
δ=ρgL2/2E (okay!)
= EL + σ1
= 62906.85 + 1804.84
= 64 711.69 psi

5. Ultimate Strength = 73 890.58 psi

6. Rupture Strength = 67 899.45 psi

Given the total mass M


Slope of 0.2% offset = E = 10,529,861.82 psi ρ=M/V=M/AL
Problem 205
A uniform bar of length L, cross-sectional δ=ρgL22E=(M/AL⋅gL2)/2E
Test for location: area A, and unit mass ρ is suspended
slope = rise / run vertically from one end. Show that its total δ=MgL2AE (okay!)
10,529,861.82 = (6989.64 + 4992.61) / run elongation is δ = ρgL2/2E. If the total mass
run = 0.00113793 of the bar is M, show also that δ = MgL/2AE. Another Solution:
Slope of EL to YP Solution 205
σ1 / ε1 = 6989.64/0.0025 δ=PL/AE
σ1 / ε1 = 2 795 856 δ=PL/AE
ε1 = σ1 / 2 795 856 From the figure:
Where:
δ=dδ P=W=(ρAL)g
For the required point:
E = (4992.61 + σ1) / ε1 P=Wy=(ρAy)g L=1/2L

10 529 861.82 = (4992.61 + σ1) / (σ1 / 2 795 L=dy Thus,


856)
3.7662 σ1 = 4992.61 + σ1 δ={[(ρAL)g](1/2L)}/AE
Thus,
δ=ρgL2/2E (okay!)
dδ=[(ρAy)gdy]/AE
28

For you to Elongation due to E = 200 000 MPa


feel the its own weight:
situation, δ1=PL/AE
Thus,
position
δ2=[20000(150000)]/[300(200000)]
yourself in
pull-up δ2=50 mm
exercise
with your
hands on Where: Total elongation:
the bar and your body hang freely above P=W= δ=δ1+δ2
the ground. Notice that your arms suffer all 7850(1/1000)3(9.81)[300(150)(1000)]
δ=4.33+50=54.33 mm answer
your weight and your lower body fells no P = 3465.3825 N
stress (center of weight is approximately L = 75(1000) = 75 000 mm Problem 207
just below the chest). If your body is the A = 300 mm2 A steel wire 30 ft long, hanging vertically,
bar, the elongation will occur at the upper E = 200 000 MPa supports a load of 500 lb. Neglecting the
half of it. weight of the wire, determine the required
diameter if the stress is not to exceed 20 ksi
Thus,
and the total elongation is not to exceed
δ1=[3465.3825(75000)]/300(200000)
Problem 206 0.20 in. Assume E = 29 × 106 psi.
A steel rod having a cross-sectional area of δ1=4.33 mm
300 mm2 and a length of 150 m is Solution 207
suspended vertically from one end. It Elongation due to
supports a tensile load of 20 kN at the lower applied load:
end. If the unit mass of steel is 7850
kg/m3 and E = 200 × 103 MN/m2, find the δ2=PL/AE
total elongation of the rod.
Where:
Solution 206 P = 20 kN = 20 000 N
L = 150 m = 150 000
mm
A = 300 mm2
29

Based on maximum F=2T


allowable stress:
p(1500)(80)=2(53333.33)
σ=P/A
p=0.8889MPa → internal pressure
2
20000=500/(1/4πd )

d=0.1784in Total normal force, N:


N = p × contact area between tire and
δ=PL/AE wheel
Based on maximum
N = 0.8889 × π(1500.5)(80)
allowable
N = 335 214.92 N
deformation: Where:
δ=PL/AE δ = π (1500.5 - 1500) = 0.5π mm Friction resistance, f:
P=T f = μN = 0.30(335 214.92)
0.20=500(30×12)/[1/4πd2(29×106)]
L = 1500π mm f = 100 564.48 N = 100.56 kN
d=0.1988in A = 10(80) = 800 mm2 Torque = f × ½(diameter of wheel)
E = 200 000 MPa Torque = 100.56 × 0.75025
Torque = 75.44 kN · m
Use the bigger diameter, d = 0.1988
inch. answer Thus,
0.5π=T(1500π)/800(200000) Problem 209
T=53333.33N An aluminum bar having a cross-sectional
Problem 208 area of 0.5 in2 carries the axial loads applied
A steel tire, 10 mm thick, 80 mm wide, and at the positions shown in Fig. P-209.
1500.0 mm inside diameter, is heated and Compute the total change in length of the
shrunk onto a steel wheel 1500.5 mm in bar if E = 10 × 106 psi. Assume the bar is
diameter. If the coefficient of static friction suitably braced to prevent lateral buckling.
is 0.30, what torque is required to twist the
tire relative to the wheel? Neglect the
deformation of the wheel. Use E = 200 GPa.

Solution 208
30

P1 = 4000 lb compression and Ebr = 83 GPa.


P2 = 11000 lb compression
Solution 209
P3 = 6000 lb compression
P1 = 6000 lb tension
P2 = 1000 lb compression
P3 = 4000 lb tension

Solution 211

δ=PL/AE

δ=−δ1−δ2−δ3

δ=[−4000(3×12)/0.5(10×106)]−[11000(5×12
δ=PL/AE
)/0.5(10×106)]−[6000(4×12)/0.5(10×106)]
Based on allowable stresses:
δ=0.2184 in (shortening) answer
δ=δ1−δ2+δ3 Steel:
δ=[6000(3×12)/0.5(10×106)]−[1000(5×12)/ Pst=σstAst
0.5(10×106)]+[4000(4×12)/0.5(10×106)] Problem 211
A bronze bar is fastened between a steel P=140(480)=67200N
δ=0.0696 in. (lengthening) answer
bar and an aluminum bar as shown in Fig. p- P=67.2kN
211. Axial loads are applied at the positions
indicated. Find the largest value of P that
Problem 210 will not exceed an overall deformation of
Solve Prob. 209 if the points of application Bronze:
3.0 mm, or the following stresses: 140 MPa
of the 6000-lb and the 4000-lb forces are Pbr=σbrAbr
in the steel, 120 MPa in the bronze, and 80
interchanged. MPa in the aluminum. Assume that the
2P=120(650)=78000N
assembly is suitably braced to prevent
Solution 210 buckling. Use Est = 200 GPa, Eal = 70 GPa, P=39000N=39kN
31

of end C must not exceed 0.10 in. Based on movement at C:


δst/2=0.1/5
Aluminum:
δst=0.04in
Pal=σalAal
PstL/AE=0.04
2P=80(320)=25600N
Pst(4×12)/[0.50(29×106)]=0.04
P=12800N=12.8kN
Pst=12083.33lb

Based on allowable deformation:


ΣMA=0
(steel and aluminum lengthens, bronze
shortens) 5P=2Pst

δ=δst−δbr+δal Solution 212 P=0.4Pst

3=P(1000)/[480(200000)]−2P(2000)/[650(8 Based on maximum stress of steel rod: P=0.4(12083.33)


3000)]+2P(1500)/[320(70000)] ΣMA=0
P=4833.33lb=4.83kips
3=(1/96000−2/26975+3/22400)P 5P=2Pst

P=42733.52N=42.73kN P=0.4Pst Use the smaller value, P = 4.83 kips


P=0.4σatAst Problem 213
Use the smallest value of P, P = 12.8 kN P=0.4[30(0.50)] The rigid bar AB, attached to two vertical
rods as shown in Fig. P-213, is horizontal
P=6kips before the load P is applied. Determine the
vertical movement of P if its magnitude is
50 kN.
Problem 212
The rigid bar ABC shown in Fig. P-212 is
hinged at A and supported by a steel rod at
B. Determine the largest load P that can be
applied at C if the stress in the steel rod is
limited to 30 ksi and the vertical movement
32

For steel: vertical movement is limited to 5 mm.


ΣMA=0 Neglect the weights of all members.

6Pst=3.5(50)

Pst=29.17kN

δ=PL/AE

δst=29.17(4)10002/300(200000)

δst=1.94mm
Solution 213

Free body diagram: Movement diagram:

Solution 41
y/3.5=1.94−1.78/6
Member AB:
y=0.09mm

δB=vertical movement of P
For aluminum:
ΣMB=0 δB=1.78+y=1.78+0.09

6Pal=2.5(50) δB=1.78+y=1.7

Pal=20.83kN δB=1.87mm answer

δ=PL/AE Problem 214


δal=20.83(3)10002/500(70000) The rigid bars AB and CD shown in Fig. P-
214 are supported by pins at A and C and ΣMA=0
δal=1.78mm the two rods. Determine the maximum 3Pal=6Pst
force P that can be applied as shown if its
33

Pal=2Pst δD=(11/42000)Pst

ΣMC=0

By ratio and proportion: 6Pst=3P

δB/6=δal/3 Pst=1/2P

δB=2δal=2[PL/AE]al
By ratio and proportion:
δB=2[Pal(2000)/500(70000)]
δP/3=δD/6
δB=1/8750Pal=1/8750(2Pst) Solution 215
δP=1/2δD=1/2(11/42000Pst)
δB=1/4375Pst → movement of B
δP=11/84000Pst

5=11/84000(1/2P)
Member CD:
P=76363.64N=76.4kN answer
ΣMal=0

Problem 215 6Pst=2W


A uniform concrete slab of total weight W is Pst=1/3W
to be attached, as shown in Fig. P-215, to
two rods whose lower ends are on the same
level. Determine the ratio of the areas of ΣMst=0
the rods so that the slab will remain level. 6Pal=4W

Pal=2/3W
Movement of D:
δD=δst+δB=[PL/AE]st+(1/4375)Pst
δst=δal
δD=Pst(2000)/300(200000)+1/4375Pst
[PL/AE]st=[PL/AE]al
34

[1/3W(6×12)]/[Ast(29×106)]=[2/3W(4×12)]/ From FBD of Joint B δBC=[6000(6×12)]/[0.6(10×106)]=0.072inch


[Aal(10×106)] ΣFH=0 shortening answer

Aal/Ast=[2/3W(4×12)(29×106)]/[1/3W(6×12)
(10×106)]

Aa/lAst=3.867 answer
PABcos30∘=PBCcos30∘

Problem 216 PAB=PBC


As shown in Fig. P-216, two aluminum rods
AB and BC, hinged to rigid supports, are ΣFV=0
pinned together at B to carry a vertical load From 'Movement of B' diagram:
P = 6000 lb. If each rod has a cross-sectional PABsin30∘+PBCsin30∘=6000 DB = δAB = 0.12 inch
area of 0.60 in.2 and E = 10 × 106 psi, PAB(0.5)+PAB(0.5)=6000 BE = δBE = 0.072 inch
compute the elongation of each rod and the δB = BB' = displacement of B
horizontal and vertical displacements of PAB=6000lb tension B' = final position of BB after elongation
point B. Assume α = 30° and θ = 30°.
PBC=6000lb compression Triangle BDB':
cosβ=0.12/δB
δ=PL/AE δB=0.12/cosβ
δAB=[6000(10×12)]/[0.6(10×106)]=0.12inch
lengthening answer
Triangle BEB':
cos(120∘−β)=0.072/δB

δB=0.072cos(120∘−β)

δB=δB

Solution 216 0.12/cosβ=0.072/cos(120∘−β)

(cos120∘cosβ+sin120∘sinβ)/cosβ=0.6

−0.5+sin120∘tanβ=0.6
35

tanβ=1.1/sin120∘ Solution 217 δ=PL/AE

β=51.79∘ δAB=[5379.45(10×12)]/[0.6(29×106)]=0.037
1 inch (lengthening)

ϕ=90−(30∘+β)=90∘−(30∘+51.79∘) δBC=[4392.30(6×12)]/[0.6(10×106)]=0.0527
inch (shortening)
ϕ=8.21∘

δB=0.12/cos51.79∘
From "Movement of B" diagram
δB=0.194inch DB = δAB = 0.0371 inch
BE = δBE = 0.0527 inch
δB = BB' = displacement of B
Triangle BFB': B' = final position of B after deformation
δh=B′F=δBsinϕ=0.194sin8.21∘

δh=0.0277inch By Sine Law Triangle BDB':


δh=0.0023ft → horizontal displacement PAB/sin60∘=6000/sin75∘ cosβ=0.0371/δB
of B answer PAB=5379.45lb (Tension) δB=0.0371/cosβ

δv=B’F=δBcosϕ=0.194cos8.21∘

δv=0.192inch Triangle BEB':


δv=0.016ft → vertical displacement of cos(105∘−β)=0.0527/δB
B answer δB=0.0527/cos(105∘−β)

Problem 217 δB=δB


Solve Prob. 216 if rod AB is of steel, with E 0.0371/cosβ=0.0527/cos(105∘−β)
= 29 × 106 psi. Assume α = 45° and θ = 30°; PBC/sin45∘=6000sin75∘
all other data remain unchanged. (cos105∘cosβ+sin105∘sinβ)/cosβ=1.4205
PBC=4392.30 lb (Compression)
−0.2588+0.9659tanβ=1.4205
36

tanβ=1.4205+0.2588/0.9659 dP = ρAω2 x dx

tanβ=1.7386 Problem 218


A uniform slender rod of length L and cross dδ=(ρAω2xdx)x/AE
β=60.1∘
sectional area A is rotating in a horizontal
δ=ρω2/E∫0Lx2dx=ρω2/E[x3/3]0L
plane about a vertical axis through one end.
δB=0.0371cos60.1∘ If the unit mass of the rod is ρ, and it is δ=ρω2/E[L3−03]
rotating at a constant angular velocity
δB=0.0371cos⁡60.1∘ of ω rad/sec, show that the total elongation δ=ρω2L3/3E okay!

δB=0.0744inch of the rod is ρω2 L3/3E.

SHEARING DEFORMATION
Solution 218
Problem 222
ϕ=(45∘+β)−90∘ δ=PL/AE A solid cylinder of diameter d carries an
axial load P. Show that its change in
ϕ=(45∘+60.1∘)−90∘
diameter is 4Pν / πEd.
ϕ=15.1∘
Solution 222
Triangle BFB':
δh=FB′=δBsinϕ=0.0744sin15.1∘

δh=0.0194inch

δh=0.00162 ft → horizontal
displacement of B answer from the frigure:
dδ=dPx/AE

δv=BF=δBcosϕ=0.0744cos15.1∘

δv=0.07183inch ν=−(εy/εx)
Where:
δv=0.00598ft → vertical displacement of εy=−νεx
dP = centrifugal force of differential mass
B answer dP = dM ω2 x = (ρA dx)ω2 x εy=−ν(σx/E)
37

δyd=−ν(−P/AE) εy is negative, thus, tensile force is required


in the x-direction to produce the same
δy=Pd/(1/4πd2E)
deformation in the y-direction as the
δy=4Pν/πEd (okay!) original forces.

For equivalent single force in the x-


Problem 223 direction:
A rectangular steel block is 3 inches long in (uniaxial stress)
the x direction, 2 inches long in the y ν=−εy/εx
direction, and 4 inches long in the z
direction. The block is subjected to a triaxial −νεx=εy
loading of three uniformly distributed
−ν(σx/E)=εy
forces as follows: 48 kips tension in the x
direction, 60 kips compression in the y −0.30(σx/29×106)=−3.276×10−4
For triaxial deformation (tensile triaxial
direction, and 54 kips tension in the z
stresses): σx=31666.67psi
direction. If ν = 0.30 and E = 29 × 106 psi,
(compressive stresses are negative stresses)
determine the single uniformly distributed σx=Px/4(2)=31666.67
εy=1/E[σy−ν(σx+σz)]
load in the x direction that would produce
the same deformation in the y direction as Px=253333.33lb (tension)
the original loading. σx=Px/Ayz=48/4(2)=6.0 ksi (tension) Px=253.33 kips (tension) answer
σy=Py/Axz=60/4(3)=5.0 ksi (compression) Problem 224
Solution 223 For the block loaded triaxially as described
σz=Pz/Axy=54/2(3)=9.0ksi (tension)
in Prob. 223, find the uniformly distributed
load that must be added in the x direction
to produce no deformation in the z
Thus,
direction.
εy=1/29×106[−5000−0.30(6000+9000)]
Solution 224
εy=−3.276×10−4
εz=1/E[σz−ν(σx+σy)]
38

Where σadded = 29000 psi σy=90MPa


σx = 6.0 ksi (tension)
Padded/2(4)=29000 psi
σy = 5.0 ksi (compression)
σz = 9.0 ksi (tension) Padded=232000lb

Padded=232 kips answer


εz=1/29×106[9000−0.3(6000−5000)]

εz=2.07×10−5
Problem 225
A welded steel cylindrical drum made of a
εz is positive, thus positive stress is needed 10-mm plate has an internal diameter of
in the x-direction to eliminate deformation 1.20 m. Compute the change in diameter εx=σx/E−ν(σy/E)
in z-direction. that would be caused by an internal εx=90/200000−0.3(45/200000)
pressure of 1.5 MPa. Assume that Poisson's
ratio is 0.30 and E = 200 GPa. εx=3.825×10−4
The application of loads is still
simultaneous:
(No deformation means zero strain) Solution 225 εx=ΔD/D
εz=1/E[σz−ν(σx+σy)]=0
σy = longitudinal stress ΔD=εxD=(3.825×10−4)(1200)
σz=ν(σx+σy) σy=pD/4t=1.5(1200)/4(10) ΔD=0.459 mm answer
σy=45MPa
Where
σy = 5.0 ksi (compression) σx = tangential stress Problem 226
σσz = 9.0 ksi (tension) σy=pD/2t=1.5(1200)/2(10) A 2-in.-diameter steel tube with a wall
thickness of 0.05 inch just fits in a rigid hole.
Find the tangential stress if an axial
9000=0.30(σx−5000) compressive load of 3140 lb is applied.
Assume ν = 0.30 and neglect the possibility
σx=35000psi of buckling.

σadded+6000=35000 Solution 226


39

ends, is 80 mm in diameter and has a wall just fits between two rigid end walls.
thickness of 3 mm. It fits without clearance Calculate the longitudinal and tangential
in an 80-mm hole in a rigid block. The tube stresses for an internal pressure of 6000 psi.
is then subjected to an internal pressure of Assume ν = 1/3 and E = 12 × 106 psi.
4.00 MPa. Assuming ν = 1/3 and E = 83 GPa,
determine the tangential stress in the tube.
Solution 228

ε=σx/E−ν(σy/E)=0

σx=νσy=σl → longitudinal stress

σt=σy → tangential stress

εx=σx/E−ν(σy/E)=0 Solution 227


σt=pD/2t=6000(3)/2(0.10)
σx=νσy Longitudinal stress:
σt=90000 psi answer
σy=pD/4t=4(80)/4(3)

Where σy=80/3MPa
σl=νσy=1/3(90000)
σx = tangential stress
σy = longitudinal stress σl=30000 psi answer
The strain in the x-direction is:
σy = Py / A = 3140 / (π × 2 × 0.05)
εx=σx/E−ν(σy/E)=0
σy = 31,400/π psi
σx=νσy → tangential stress
Thus, σx=1/3(80/3)
σx=0.30(31400/π)
σx=8.89 MPa answer
σx=9430/π psi

σx=2298.5 psi answer


Problem 228
A 6-in.-long bronze tube, with closed ends,
is 3 in. in diameter with a wall thickness of
Problem 227
0.10 in. With no internal pressure, the tube
A 150-mm-long bronze tube, closed at its
40

δ=δcastiron=δsteel=0.8mm Problem 234


A reinforced concrete column 200 mm in
diameter is designed to carry an axial
δcastiron=Pcastiron(2000)/[1/4π(602−502)](1000
compressive load of 300 kN. Determine the
00)=0.8
required area of the reinforcing steel if the
Pcastiron=11000πN allowable stresses are 6 MPa and 120 MPa
for the concrete and steel, respectively. Use
Eco = 14 GPa and Est = 200 GPa.
δsteel=Psteel(2000)/[1/4π(502)](200000)=0.8

STATICALLY INDETERMINATE Psteel=50000πN Solution 234

δco=δst=δ
Problem 233 (PL/AE)co=(PL/AE)st
A steel bar 50 mm in
diameter and 2 m (σL/E)co=(σL/E)st
long is surrounded σcoL/14000=σstL/200000
by a shell of a cast
iron 5 mm thick. 100σco=7σst
Compute the load
that will compress
the combined bar a
total of 0.8 mm in Psteel=50000πN
the length of 2 m.
For steel, E = 200 ΣFV=0
GPa, and for cast P=Pcastiron+Psteel
iron, E = 100 GPa.
P=11000π+50000π

Solution 233 P=61000πN

δ=PL/AE P=191.64kN answer


41

What will be the stress at 0°F? At what


temperature will the stress be zero?
Assume α = 6.5 × 10-6 in/(in·°F) and E = 29 ×
When σst = 120 MPa 106 psi.
100σco=7(120)

σco=8.4 MPa>6 MPa → (not okay!) Solution 261


δT=δst
For the stress at 0°F:
When σco = 6 MPa αL(ΔT)=PL/AE
100(6)=7σst
α(ΔT)=P/AE
σst=85.71 MPa<120 MPa → (okay!)
(6.5×10−6)(T−70)=1200/0.25(29×106)

T=95.46∘F answer
Use σco = 6 MPa and σst = 85.71 MPa.

ΣFV=0 Problem 262


δ=δT+δst A steel rod is stretched between two rigid
Pst+Pco=300
walls and carries a tensile load of 5000 N at
σL/E=αL(ΔT)+PL/AE
σstAst+σcoAco=300 20°C. If the allowable stress is not to exceed
σ=αE(ΔT)+P/A 130 MPa at -20°C, what is the minimum
85.71Ast+6[1/4π(200)2−Ast]=300(1000) diameter of the rod? Assume α =
σ=(6.5×10−6)(29×106)(70)+1200/0.25 11.7 µm/(m·°C) and E = 200 GPa.
79.71Ast+60000π=300000
σ=17995psi=18 ksi answer
Ast=1398.9 mm2 answer
Solution 262

For the temperature that causes zero


THERMAL
stress:
Problem 261
A steel rod with a cross-sectional area of
0.25 in2 is stretched between two fixed
points. The tensile load at 70°F is 1200 lb.
42

δ=δT+δst δT=αL(Tf−Ti) A drop of temperature is needed to


increase the stress to 10 ksi. See figure
σL/E=αL(ΔT)+PL/AE 3=(11.7×10−6)(10000)(Tf−15)
below.
σ=αE(ΔT)+P/A Tf=40.64∘C answer

130=(11.7×10−6)(200000)(40)+5000/A
Required stress:
A=5000/36.4=137.36 mm2
δ=δT

σL/E=αL(ΔT)
1/4πd2=137.36
σ=αE(Tf−Ti)
d=13.22 mm answer
σ=(11.7×10−6)(200000)(40.64−15) δ=δT+δst
Problem 263
Steel railroad reels 10 m long are laid with a σ=60 MPa answer σL/E=αL(ΔT)+PL/AE
clearance of 3 mm at a temperature of σ=αE(ΔT)+P/A
Problem 264
15°C. At what temperature will the rails just
A steel rod 3 feet long with a cross-sectional 10000=(6.5×10−6)(29×106)(ΔT)+1200/0.25
touch? What stress would be induced in the
area of 0.25 in.2 is stretched between two
rails at that temperature if there were no
fixed points. The tensile force is 1200 lb at ΔT=27.59∘F
initial clearance? Assume α =
40°F. Using E = 29 × 106 psi and α = 6.5 × 10-
11.7 µm/(m·°C) and E = 200 GPa. 6
in./(in.·°F), calculate (a) the temperature
Required temperature: (temperature must
at which the stress in the bar will be 10 ksi;
drop from 40°F)
Solution 263 and (b) the temperature at which the stress
will be zero. T=40−27.59=12.41∘F answer

Solution 264
(b) Temperature at which the stress will be
(a) Without temperature change: zero:

σ=P/A=1200/0.25=4800 psi From the figure below:

Temperature at which δT = 3 mm: σ=4.8ksi<10ksi


δT=αL(ΔT)
43

Solution 265

δT=δ+Δ

δ=δT

PL/AE=αL(ΔT)

P=αAE(Tf−Ti) αL(ΔT)=σL/E+2.5

(18×10−6)(3000)(ΔT)=35(3000)/80000+2.5 Solution 266


1200=(6.5×10−6)(0.25)(29×106)(Tf−40)
ΔT=70.6∘C δT=αLΔT
Tf=65.46∘F answer
T=70.6−20

Problem 265 T=50.6∘C answer


A bronze bar 3 m long with a cross sectional
area of 320 mm2 is placed between two
rigid walls as shown in Fig. P-265. At a Problem 266
temperature of -20°C, the gap Δ = 2.5 mm. Calculate the increase in stress for each
segment of the compound bar shown in Fig. δT(st)=(6.5×10−6)(15)(100)
Find the temperature at which the
compressive stress in the bar will be 35 P-266 if the temperature increases by δT(st)=0.00975
MPa. Use α = 18.0 × 10-6 m/(m·°C) and E = 100°F. Assume that the supports are
80 GPa. unyielding and that the bar is suitably
braced against buckling. δT(al)=(12.8×10−6)(10)(100)

δT(al)=0.0128in

δst+δal=δT(st)+δT(al)

(PL/AE)st+(PL/AE)al=0.00975+0.0128
44

Where Pst = Pal = P. Thus, P=(11.7×10−6)(80−25)(90×12)(200000)


P(15)/1.5(29×106)+P(10)/2(10×106)=0.022
P=138996N
55

P=26691.84lb

σ=P/A

σst=26691.841.5=17794.56 psi answer

σal=26691.84/2.0=13345.92 psi answ


er Solution 268

Contraction of steel rod, assuming


F=2P complete freedom:
Problem 267
At a temperature of 80°C, a steel tire 12 mm δT(st)=αLΔT
pDL=2P
thick and 90 mm wide that is to be shrunk δT(st)=(11.7×10−6)(900)(40)
onto a locomotive driving wheel 2 m in p(2000)(90)=2(138996)
diameter just fits over the wheel, which is δT(st)=0.4212mm
p=1.5444 MPa answer
at a temperature of 25°C. Determine the The steel rod cannot freely contract
contact pressure between the tire and because of the resistance of aluminum rod.
wheel after the assembly cools to 25°C. Problem 268 The movement of A (referred to as δA),
Neglect the deformation of the wheel The rigid bar ABC in Fig. P-268 is pinned at therefore, is less than 0.4212 mm. In terms
caused by the pressure of the tire. B and attached to the two vertical rods. of aluminum, this movement is (by ratio
Assume α = 11.7 μm/(m·°C) and E = 200 Initially, the bar is horizontal and the and proportion):
GPa. vertical rods are stress-free. Determine the
stress in the aluminum rod if the
δA/0.6=δal/1.2
Solution 267 temperature of the steel rod is decreased
by 40°C. Neglect the weight of bar ABC. δA=0.5δal
δ=δT

PL/AE=αLΔT

P=αΔTAE
45

slab that is supported by two copper bars.


At 10°C, Δ = 0.18 mm. Neglecting the mass
of the slab, calculate the stress in each rod
when the temperature in the assembly is
increased to 95°C. For each copper bar, A =
500 mm2, E = 120 GPa, and α =
16.8 µm/(m·°C). For the aluminum bar, A =
δT(st)−δst=0.5δal 400 mm2, E = 70 GPa, and α =
23.1 µm/(m·°C).
0.4212−(PL/AE)st=0.5(PL/AE)al

0.4212−Pst(900)/300(200000)=0.5[Pal(1200 From the figure:


)/1200(70000)] δT(al)−δal−Δ=δT(co)+δco
28080−Pst=0.4762Pal → Equation (1) 1.472−(PL/AE)al−0.18=1.071+(PL/AE)co

1.472−2F(750−0.18)/400(70000)−0.18=1.0
ΣMB=0 71+F(750)/500(120000)
0.6Pst=1.2Pal 0.221=(6.606×10−5)F
Pst=2Pal → Equation (2) F=3345.44N

Pco=F=3345.44N
Equations (1) and (2) Solution 269
28080−2Pal=0.4762Pal Pal=2F=6690.89N
Assuming complete freedom:
Pal=11340N δT=αLΔT σ=P/A
−6
δT(co)=(16.8×10 )(750)(95−10) σco=3345.44/500=6.69 MPa answer
σal=Pal/Aal=11340/1200
δT(co)=1.071mm σal=6690.89/400=16.73 MPa answer
σal=9.45 MPa answer
−6
δT(al)=(23.1×10 )(750−0.18)(95−10)
Problem 269
As shown in Fig. P-269, there is a gap δT(al)=1.472mm
between the aluminum bar and the rigid
46

Problem 271
A rigid bar of negligible weight is supported
as shown in Fig. P-271. If W = 80 kN,
compute the temperature change that will
cause the stress in the steel rod to be 55
MPa. Assume the coefficients of linear
expansion are 11.7 µm/(m·°C) for steel and By ratio and proportion:
18.9 µm/(m·°C) for bronze. δT(st)+δst/1=δT(br)+δbr/4

δT(st)+δst=0.25[δT(br)+δbr]

(αLΔT)st+(σL/E)st=0.25[(αLΔT)br+(σL/E)br]
(11.7×10−6)(1500)ΔT+55(1500)/2000=0.25[ Solution 272
(18.9×10−6)(3000)ΔT+35.08(3000)/83000]
ΣMA=0
0.01755ΔT+0.4125=0.014175ΔT+0.317
4Pbr+Pst=2.5(80000)
ΔT=−28.3∘C
4σbr(1300)+σst(320)=2.5(80000)
A temperature drop of 28.3°C is needed to
stress the steel to 55 MPa. answer 16.25σbr+σst=625

σst=625−16.25σbr Equation (1)


Solution 271
Problem 272
Stress in bronze when σst = 55 MPa For the assembly in Fig. 271, find the stress
ΣMA=0 in each rod if the temperature rises 30°C
after a load W = 120 kN is applied.
4Pbr+Pst=2.5(80000)

4σbr(1300)+55(320)=2.5(80000)

σbr=35.08MPa

δT(st)+δst/1=δT(br)+δbr/4
47

δT(st)+δst=0.25[δT(br)+δbr] aluminum. δT(st)=0.0039inch

(αLΔT)st+(σL/E)st=0.25[(αLΔT)br+(σL/E)br] δT(al)−δal=δst−δT(st)

0.01152−(PL/AE)al=(PL/AE)st−0.0039
−6
(11.7×10 )(1500)(30)+σst(1500)/200000=0
0.01152−R(15)/2(10×106)=(R+50000)(10)/3
.25[(18.9×10−6)(3000)(30)+σbr(3000)/8300
(29×106)−0.0039
0]
100224−6.525R=R+50000−33930
0.5265+0.0075σst=0.42525+0.00904σbr
84154=7.525R
0.0075σst−0.00904σbr=−0.10125
R=11183.25lbs
0.0075(625−16.25σbr)−0.00904σbr=−0.1012
5
Solution 273 Pal=R=11183.25lbs
4.6875−0.121875σbr−0.00904σbr=−0.10125
δT(al)=(αLΔT)al Pst=R+50000=61183.25lbs
4.78875=0.130915σbr

σbr=36.58 MPa answer δT(al)=(12.8×10−6)(15)(120−60)


σ=P/A
δT(al)=0.01152inch
σal=11183.252=5591.62psi answer
σst=625−16.25(36.58)
σst=61183.253=20394.42psi answer
σst=30.58 MPa answer

Problem 274
Problem 273
At what temperature will the aluminum and
The composite bar shown in Fig. P-273 is
steel segments in Prob. 273 have
firmly attached to unyielding supports. An
numerically equal stress?
axial force P = 50 kips is applied at 60°F.
Compute the stress in each material at
120°F. Assume α = 6.5 × 10-6 in/(in·°F) for Solution 274
δT(st)=(αLΔT)st
steel and 12.8 × 10-6 in/(in·°F) for
σal=σst
δT(st)=(6.5×10−6)(10)(120−60)
48

R1/2=(50000−R1)/3 A drop of 44.94°F from the standard 5σbrAbr−3σcoAco=0


temperature will make the aluminum and
3R1=100000−2R1 5(90)(1200)−3σco(1500)=0
steel segments equal in stress. answer
R1=20000lbs σco=120MPa

Problem 275
A rigid horizontal bar of negligible mass is
connected to two rods as shown in Fig. P-
275. If the system is initially stress-free.
Calculate the temperature change that will
cause a tensile stress of 90 MPa in the brass
rod. Assume that both rods are subjected to
the change in temperature.

δ=PL/AE δ=σL/E

δal=20000(15)/2(10×106)=0.015inch δbr=90(2000)100000=1.8mm

δst=(50000−20000)(10)/3(29×106)=0.0034 δco=120(3000)120000=3mm
5inch
By ratio and proportion
δT(co)−δco/3=δbr−δT(br)/5
δal−δT(al)=δst+δT(st)
5δT(co)−5δco=3δbr−3δT(br)
−6
0.015−(12.8×10 )(15)ΔT=0.00345+(6.5×10
−6
5(16.8×10−6)(3000)ΔT−5(3)=3(1.8)−3(18.7×
)(10)ΔT
10−6)(2000)ΔT
0.01155=0.000257ΔT
0.3642ΔT=20.4
ΔT=44.94∘F Solution 275
ΔT=56.01∘C drop in
ΣMhingesupport=0 temperature answer
5Pbr−3Pco=0
49

Problem 276 (70,200+P1)1.2247L2=0.8165(70,200+P2)L2


Four steel bars jointly support a mass of 15
1.5(70,200+P1)=70,200+P2
Mg as shown in Fig. P-276. Each bar has a
cross-sectional area of 600 mm2. Find the P2=1.5P1+35,100 → Equation (1)
load carried by each bar after a
temperature rise of 50°C. Assume α =
11.7 µm/(m·°C) and E = 200 GPa. ΣFV=0

h=h

L1sin45∘=L2sin60∘ 2(P1sin45∘)+2(P2sin60∘)=147.15(1000)
L1=1.2247L2 P1sin45∘+P2sin60∘=72,575
δ1=δsin45∘ P1sin45∘+(1.5P1+35,100)sin60∘=72,575
δ2=δsin60∘ 0.7071P1+1.299P1+30,397.49=72,575

2.0061P1=42,177.51
Solution 276
δ1/δ2=δsin45∘/δsin60∘ P1=21,024.63N
h=L1sin45∘
δ1=0.8165δ2
h=L2sin60∘ P2=1.5(21,024.63)+35,100
αL1ΔT+P1L1/AE=0.8165[αL2ΔT+P2L2/AE] P2=66,636.94N

(11.7×10−6)L1(50)+P1L1/600(200000)=0.81 PA=PD=P1=21.02kN answer


65[(11.7×10−6)(50)+P2L2/600(200000)]
PB=PC=P2=66.64kN answer
70,200L1+P1L1=0.8165(70,200L2+P2L2)

(70,200+P1)L1=0.8165(70,200+P2)L2
TORSION
50

TORSION 3∘(π/180∘)=12(6)(10003)/[1/32πd4(83000)] Solution 307

Problem 304 d=113.98mm answer θ=TL/JG


A steel shaft 3 ft long that has a diameter of
4∘(π/180∘)=T(5)(1000)/[1/32πd4(83000)]
4 in is subjected to a torque of 15 kip·ft.
τmax=16T/πd3=16(12)(10002)/π(113.983)
Determine the maximum shearing stress T=0.1138d4
and the angle of twist. Use G = 12 × 106 psi. τmax=41.27MPa answer

Problem 306 τmax=16T/πd3


Solution 304 A steel marine propeller shaft 14 in. in
80=16(0.1138d4)/πd3
3 3 diameter and 18 ft long is used to transmit
τmax=16T/πD =16(15)(1000)(12)/π(4 )
5000 hp at 189 rpm. If G = 12 × 106 psi, d=138 mm answer
τmax=14324psi determine the maximum shearing stress.

τmax=14.3ksi answer T=P/2πf


Solution 306
0.1138d4=P/2π(20)
θ=TL/JG=15(3)(1000)(122)/[1/32π(44)(12× Note: 1 hp = 33,000 ft·lb/min = 396,000
106)] lb·in/min P=14.3d4=14.3(1384)
T=P/2πf=5000(39600)/2π(189) P=5186237285N⋅mm/sec
θ=0.0215rad
T=1667337.5lb⋅in P=5186237.28W
θ=1.23∘ answer

Problem 305 P=5.19 MW answer


τmax=16T/πd3=16(1667337.5)/π(143)
What is the minimum diameter of a solid Problem 308
steel shaft that will not twist through more τmax=3094.6psi answer A 2-in-diameter steel shaft rotates at 240
than 3° in a 6-m length when subjected to a rpm. If the shearing stress is limited to 12
Problem 307
torque of 12 kN·m? What maximum ksi, determine the maximum horsepower
A solid steel shaft 5 m long is stressed at 80
shearing stress is developed? Use G = 83 that can be transmitted.
MPa when twisted through 4°. Using G = 83
GPa.
GPa, compute the shaft diameter. What
power can be transmitted by the shaft at 20 Solution 308
Solution 305 Hz?
τmax=16T/πd3
θ=TL/JG
51

12(1000)=16T/π(23) 1∘(π/180∘)=238732.41(26d)(1000)/[1/32πd τmax−solid=15/16[162T/15πD3]


4
(83000)]
T=18849.56lb⋅in τmax−solid=15/16×τmax−hollow (okay!)
d=352.08mm
T=P/2πf Problem 311
An aluminum shaft with a constant
18849.56=P(396000)/2π(240)
Use the larger diameter, thus, d = 352 diameter of 50 mm is loaded by torques
P=71.78 hp answer mm. answer applied to gears attached to it as shown in
Fig. P-311. Using G = 28 GPa, determine the
Problem 309 Problem 310
relative angle of twist of gear D relative to
A steel propeller shaft is to transmit 4.5 Show that the hollow circular shaft whose
gear A.
MW at 3 Hz without exceeding a shearing inner diameter is half the outer diameter
stress of 50 MPa or twisting through more has a torsional strength equal to 15/16 of
than 1° in a length of 26 diameters. that of a solid shaft of the same outside
Compute the proper diameter if G = 83 GPa. diameter.

Solution 309 Solution 310

T=P/2πf=4.5(1000000)/2π(3) Hollow circular shaft:


τmax−hollow=16TD/π(D4−d4)
T=238732.41N⋅m
τmax−hollow=16TD/π[D4−(1/2D)4]

Based on maximum allowable shearing τmax−hollow=16TD/π(15/16D4)


stress:
τmax−hollow=162T/15πD3
τmax=16T/πd3

50=16(238732.41)(1000)/πd3 Solid circular shaft:

d=289.71mm

Based on maximum allowable angle of


twist:
τmax−solid=16T/πD3
θ=TL/JG
52

relative to the other end? G = 12 × 106 psi. θ=0.5234rad=30∘ answer

Solution 311 Problem 313


Solution 312 Determine the maximum torque that can
θ=TL/JG
be applied to a hollow circular steel shaft of
100-mm outside diameter and an 80-mm
inside diameter without exceeding a
shearing stress of 60 MPa or a twist of 0.5
deg/m. Use G = 83 GPa.

τmax=16T/πd3
Solution 313
20(1000)=16T/π(0.20)3
Based on maximum allowable shearing
Rotation of D relative to A: T=10πlb⋅in stress:
θD/A=1/JG(ΣTL) τmax=16TD/π(D4−d4)
θD/A=1/[1/32π(504)(28000)][800(2)−300(3) 60=16T(100)/π(1004−804)
+600(2)](10002) L=T/(0.50lb⋅in/in)
T=6955486.14N⋅mm
θD/A=0.1106rad L=10πlb⋅in/(0.50lb⋅in/in)
T=6955.5N⋅m
θD/A=6.34∘ answer L=20πin=62.83in

Based on maximum allowable angle of


Problem 312 θ=TL/JG twist:
A flexible shaft consists of a 0.20-in-
diameter steel wire encased in a stationary θ=TL/JG
tube that fits closely enough to impose a If θ = dθ, T = 0.5L and L = dL
0.5∘(π/180∘)=T(1000)/[1/32π(1004−804)(83
frictional torque of 0.50 lb·in/in. Determine ∫dθ=1/JG∫020π(0.5L)dL 000)]
the maximum length of the shaft if the
shearing stress is not to exceed 20 ksi. What θ=[0.5L22]02π =1/JG[0.25(20π)2−0.25(0)2] T=4198282.97N⋅mm
will be the angular deformation of one end θ=100π2/(1/32π(0.204)(12×106)) T=4198.28N⋅m
53

that the shearing stress will not exceed 60


MPa. (b) If a uniform shaft diameter of 100
Use the smaller torque, T = 4 198.28
mm is specified, determine the angle by
N·m. answer
which one end of the shaft lags behind the
other end. Use G = 83 GPa.

Problem 314
The steel shaft shown in Fig. P-314 rotates Solution 315
τmax=16T/πd3
at 4 Hz with 35 kW taken off at A, 20 kW
T=P/2πf
removed at B, and 55 kW applied at C. Using τAB=16(1392.6)(1000)/π(553)=42.63MPa
G = 83 GPa, find the maximum shearing TA=TC=−20(1000)/2π(2)=−1591.55N⋅m
τBC=16(2188.4)(1000)/π(653)=40.58MPa
stress and the angle of rotation of gear A
TB=70(1000)/2π(2)=5570.42N⋅m
relative to gear C. ∴ τmax=τAB=42.63 MPa answer
TD=−30(1000)/2π(2)=−2387.32N⋅m

θ=TL/JG

θA/C=(1/G)Σ(TL/J)

θA/C=1/83000{[1392.6(4)/[1/32π(554)]+[21
88.4(2)/(1/32)π(654)]}(10002)
Solution 314
θA/C =0.104796585rad
T=P/2πf
θA/C =6.004∘ answer
TA=−35(1000)/2π(4)=−1392.6N⋅m

TB=−20(1000)/2π(4)=−795.8N⋅m
Problem 315
TC=55(1000)/2π(4)=2188.4N⋅m
A 5-m steel shaft rotating at 2 Hz has 70 kW Part (a)
applied at a gear that is 2 m from the left τmax=16T/πd3
end where 20 kW are removed. At the right
Relative to C: end, 30 kW are removed and another 20
kW leaves the shaft at 1.5 m from the right
end. (a) Find the uniform shaft diameter so
54

For AB upon by two torques as shown in Fig. P-316. T=691.15N⋅m


60=16(1591.55)(1000)/πd3 Determine the maximum permissible value
of T subject to the following conditions: τst ≤
d=51.3mm
83 MPa, τal ≤ 55 MPa, and the angle of
For BC rotation of the free end is limited to 6°. For
60=16(3978.87)(1000)/πd3 steel, G = 83 GPa and for aluminum, G = 28
GPa.
d=69.6mm

For CD
60=16(2387.32)(1000)/πd3

d=58.7mm Based on maximum angle of twist, θmax = 6°:

θ=(TL/JG)st+(TL/JG)al

Use d = 69.6 mm answer 6∘(π/180∘)=3T(900)/[1/32π(504)(83000)]+T


(600)/[1/32π(404)(28000)]
Solution 316
Part (b) T=757316.32N⋅mm
θ=TL/JG Based on maximum shearing stress, τmax =
T=757.32N⋅m
16T / πd3:
θD/A=(1/JG)ΣTL
Steel
θD/A=1/[1/32π(1004)(83000)][−1591.55(2)+
Use the least value of T. Thus, T = 679.04
3978.87(1.5)+2387.32(1.5)](10002) τst=16(3T)/π(503)=83
N·m answer
θD/A=0.007813rad T=679042.16N⋅mm
Problem 317
θD/A=0.448∘ answer T=679.04N⋅m A hollow bronze shaft of 3 in. outer
diameter and 2 in. inner diameter is slipped
over a solid steel shaft 2 in. in diameter and
Aluminum of the same length as the hollow shaft. The
Problem 316
A compound shaft consisting of a steel τal=16T/π(403)=55 two shafts are then fastened rigidly
segment and an aluminum segment is acted together at their ends. For bronze, G = 6 ×
T=691150.38N⋅mm
55

106 psi, and for steel, G = 12 × 106 psi. What Equation (1) with Tbr in terms of Tst and
torque can be applied to the composite Equation (2)
shaft without exceeding a shearing stress of T=(Tst+390×106/192×106)Tbr
8000 psi in the bronze or 12 ksi in the steel?
Tst=0.3299T

Solution 317
Based on hollow bronze (Tbr = 0.6701T)
θst=θbr τmax=[16TD/π(D4−d4)]br Solution 318

(TL/JG)st=(TL/JG)br 8000=[16(0.6701T)(3)]/π(34−24) τmax=16T/πD3

TstL/[1/32π(24)(12×106)]=TbrL/[1/32π(34−24 T=50789.32lb⋅in
)(6×106)]
T=4232.44lb⋅ft
Tst/192×106=Tbr/390×106 → Equation (1)

Based on steel core (Tst = 0.3299T):


τmax=[16TD/πD3]st

12000=16(0.3299T)/π(23)

T=57137.18lb⋅in

T=4761.43lb⋅ft

Applied Torque = Resisting Torque Use T = 4232.44 lb·ft. answer For the 2-ft segment:

T=Tst+Tbr → Equation (2) Problem 318 τmax2=16(600)(12)/π(23)=4583.66psi


A solid aluminum shaft 2 in. in diameter is answer
Equation (1) with Tst in terms of Tbr and subjected to two torques as shown in Fig. P-
Equation (2) 318. Determine the maximum shearing
6 6
T=(192×10 /390×10 )Tbr+Tbr stress in each segment and the angle of For the 3-ft segment:
rotation of the free end. Use G = 4 × 106 psi.
Tbr=0.6701T τmax3=16(800)(12)/π(23)=6111.55psi a
nswer
56

Solution 319 Tbr=4.970 kN⋅m → Maximum allowable


torque for bronze
Angle of twist ΣM=0

θ=TL/JG T=Tbr+Tst → Equation (1)


From Equation (2b)
θ=(1/JG)ΣTL Tst=0.6246(4.970)
θbr=θst
θ=1/[1/31π(24)(4×106)][600(2) Tst=3.104 kN⋅m
+800(3)](122) (TL/JG)br=(TL/JG)st

θ=0.0825rad Tbr(2)(1000)/[1/32π(754)(35000)]=Tst(1.5)(
Based on τbr ≤ 80 MPa
1000)/[1/32π(504)(83000)]
θ=4.73∘ answer
80=16Tst/π(503)
Tbr=1.6011Tst → Equation (2a)
Tst=1963495.41N⋅mm
Tst=0.6246Tbr → Equation (2b)
Problem 319
Tst=1.963 kN⋅m → Maximum allowable
The compound shaft shown in Fig. P-319 is
torque for steel
attached to rigid supports. For the bronze
segment AB, the diameter is 75 mm, τ ≤ 60
MPa, and G = 35 GPa. For the steel segment From Equation (2a)
BC, the diameter is 50 mm, τ ≤ 80 MPa, and Tbr=1.6011(1.963)
G = 83 GPa. If a = 2 m and b = 1.5 m,
Tbr=3.142 kN⋅m
compute the maximum torque T that can
be applied.
τmax=16T/πD3
From Equation (1), use

Based on τbr ≤ 60 MPa Tbr = 3.142 kN·m and Tst = 1.963 kN·m
T=3.142+1.963
60=16Tbr/π(753)
T=5.105kN⋅m answer
Tbr=4970097.75N⋅mm
Problem 320
In Prob. 319, determine the ratio of lengths
b/a so that each material will be stressed to
57

its permissible limit. What torque T is Equations (1) and (2a):


required? T=b/aT2+T2

T=(T2b+T2a)/a
Solution 320
T=(b+a)T2/a
From the solution of Problem 319:
T=LT2/a
Maximum Tbr = 4.970 kN·m
Maximum Tst = 1.963 kN·m Solution 321 T2=Ta/L (okay!)

ΣM=0
θbr=θst If the shaft were hollow, Equation (1) would
T=T1+T2 → Equation (1) be the same and the equality θ1 = θ2, by
(TL/JG)br=(TL/JG)st
direct investigation, would yield the same
4.973a(10002)/[1/32π(754)(35000)]=1.963b θ1=θ2 result in Equations (2a) and (2b). Therefore,
(10002)/[1/32π(504)(83000)] the values of T1 and T2 are the same (no
(TL/JG)1=(TL/JG)2 change) if the shaft were hollow.
b/a=1.187
T1a/JG=T2b/JG Problem 322
T1=b/aT2 → Equation (2a) A solid steel shaft is loaded as shown in Fig.
T=Tbrmax+Tstmax
P-322. Using G = 83 GPa, determine the
T=4.970+1.963 T2=a/bT1 → Equation (2b) required diameter of the shaft if the
shearing stress is limited to 60 MPa and the
T=6.933kN⋅m answer angle of rotation at the free end is not to
Equations (1) and (2b):
Problem 321 exceed 4 deg.
T=T1+a/bT1
A torque T is applied, as shown in Fig. P-
321, to a solid shaft with built-in ends. T=(T1b+T1a)/b
Prove that the resisting torques at the walls T=[(b+a)T1]/b
are T1 = Tb/L and T2 = Ta/L. How would
these values be changed if the shaft were T=LT1/b
hollow?
T1=Tb/L (okay!)
58

Solution 322 D=51.89 mm

Based on maximum allowable shear:


τmax=16T/πD3 Use D=51.89 mm answer

Problem 323
A shaft composed of segments AC, CD, and
DB is fastened to rigid supports and loaded
as shown in Fig. P-323. For bronze, G = 35
GPa; aluminum, G = 28 GPa, and for steel, G
= 83 GPa. Determine the maximum
shearing stress developed in each segment. The rotation of B relative to A is zero.
θA/B=0

Σ(TL/JG)A/B=0

TA(2)(10002)/[1/32π(254)(35000)]+
(TA−300)(2)(10002)/[1/32π(504)(28000)]+
For the 1st segment:
60=16(450)(1000)πD3 (TA−1000)(2.5)(10002)/[1/32π(254)(83000)]=0

D=33.68 mm
2TA/(254)(35)+2(TA−300)/(504)(28)+2.5(TA−
Solution 323 1000)/(254)(83)=0
For the 2nd segment:
60=16(1200)(1000)/πD3 Stress developed in each segment with 16TA/35+(TA−300)/28+[20(TA−1000)]/83=
respect to TA: 0
D=46.70 mm
(16/25)TA+(1/28)TA−75/7+(20/83)TA−200
Based on maximum angle of twist: 00/83=0
θ=TL/JG (8527/11620)TA=251.678
θ=(1/JG)ΣTL TA=342.97N⋅m
4∘(π/180∘)=1/[1/32πD4(83000)][450(2.5)+
1200(2.5)](10002) ΣM=0
59

ΣM=0 limit when a torque T = 12 kip·ft is applied.


For bronze, G = 6 × 106 psi and for steel, G
TA+TB=300+700 ΣM=0
= 12 × 106 psi.
342.97+TB=1000 Tbr+Tst=T

TB=657.03N⋅m Tbr+Tst=12(1000)(12)

Tbr+Tst=144000lb⋅in
Tbr=342.97N⋅m
500πDbr3+750πDst3=144000
Tal=342.97−300=42.97N⋅m
Dbr3=288/π+1.5Dst3 → Equation (1)
Tst=342.97−1000=−657.03N⋅m=−TB (oka
y!)
θbr=θst
Solution 324
(TLJG)br=(TLJG)st
τmax=16T/πD3 τmax=16T/πD3
Tbr(6)/[1/32πDbr4(6×106)]=Tst(4)/[1/32πDst4
τbr=16(342.97)(1000)/π(253)=111.79 MPa For bronze:
(12×106)]
answer
8000=16Tbr/πDbr3
3 Tbr/Dbr4=Tst/3Dst4
τal=16(42.97)(1000)/π(50 )=1.75 MPa
answer Tbr=500πDbr3lb⋅in
500πDbr3/Dbr4=750πDst3/3Dst4
τst=16(657.03)(1000)/π(253)=214.16 MPa Dst=0.5Dbr
answer For steel:
Problem 324 12000=16Tst/πDst3 From Equation (1)
The compound shaft shown in Fig. P-324 is Dbr =288/π−1.5(0.5Dbr)3
3

attached to rigid supports. For the bronze Tst=750πDst3lb⋅in


segment AB, the maximum shearing stress Dbr=288/π
is limited to 8000 psi and for the steel Dbr=4.26in. answer
segment BC, it is limited to 12 ksi.
Determine the diameters of each segment Dst=0.5(4.26)=2.13in. answer
so that each material will be
simultaneously stressed to its permissible
60

Problem 325 τmax=16T/πD3


The two steel shaft shown in Fig. P-325,
τof6.5'shaft=16(817.32)(12)/π(23)=6243.86 psi
each with one end built into a rigid support
answer
have flanges rigidly attached to their free
ends. The shafts are to be bolted together τof3.25'shaft=16(817.32)(12)/π(1.53)=14800.27 p
at their flanges. However, initially there is a si answer
6° mismatch in the location of the bolt holes
as shown in the figure. Determine the
maximum shearing stress in each shaft COUPLING T=1/4π(202)(40)(200)(10)
after the shafts are bolted together. Use G
Problem 326 T=8000000πN⋅mm
= 12 × 106 psi and neglect deformations of
the bolts and flanges. A flanged bolt coupling consists of ten 20- T=8πkN⋅m=25.13 kN⋅m answer
mm-diameter bolts spaced evenly around a
bolt circle 400 mm in diameter. Determine
the torque capacity of the coupling if the Problem 327
allowable shearing stress in the bolts is 40 A flanged bolt coupling consists of ten steel
MPa. ½-in.-diameter bolts spaced evenly around
a bolt circle 14 in. in diameter. Determine
Solution 326 the torque capacity of the coupling if the
allowable shearing stress in the bolts is
6000 psi.

Solution 327
Solution 325 T=PRn=AτRn=1/4πd2τRn
Θof 6.5' shaft+θof 3.25' shaft=6∘ T=1/4π(1/2)2(6000)(7)(10)
2
T=PRn=AτRn=1/4πd τRn
(TL/JG)of 6.5' shaft+(TL/JG)of 3.25' shaft=6∘(π180∘) T=26250πlb⋅in
[T(6.5)(122)]/(1/32π(24)(12×106)+T(3.25)(1 T=2187.5πlb⋅ft=6872.23 lb⋅ft answer
22)/[1/32π(1.54)(12×106)]=π30
Problem 328
T=817.32lb⋅ft A flanged bolt coupling consists of eight 10-
61

mm-diameter steel bolts on a bolt circle Problem 329 Problem 330


400 mm in diameter, and six 10-mm- A torque of 700 lb-ft is to be carried by a Determine the number of 10-mm-diameter
diameter steel bolts on a concentric bolt flanged bolt coupling that consists of eight steel bolts that must be used on the 400-
circle 300 mm in diameter, as shown in Fig. ½-in.-diameter steel bolts on a circle of mm bolt circle of the coupling described
3-7. What torque can be applied without diameter 12 in. and six ½-in.-diameter steel in Prob. 328 to increase the torque capacity
exceeding a shearing stress of 60 MPa in the bolts on a circle of diameter 9 in. Determine to 14 kN·m
bolts? the shearing stress in the bolts.

Solution 330
Solution 328 Solution 329
T=P1R1n1+P2R2n2
For one bolt in the outer circle: P1/R1=P2/R2
14(10002)=1500π(200)n1+1125π(150)(6)
2
P1=Aτ=[π(10 )/4](60) Aτ1/6=Aτ2/4.5
n1=11.48 say 12 bolts answer
P1=1500πN τ2=0.75τ1

Problem 331
T= P1R1n1 + P2R2n2
For one bolt in the inner circle: A flanged bolt coupling consists of six ½-in.
700(12)=1/4π(1/2)2τ1(6)(8)+1/4π(1/2)2τ2(4. steel bolts evenly spaced around a bolt
P1/R1=P2/R2
5)(6) circle 12 in. in diameter, and four ¾-in.
1500π/200=P2/150 aluminum bolts on a concentric bolt circle 8
8400=3πτ1+1.6875π(0.75τ1)
in. in diameter. What torque can be applied
P2=1125πN without exceeding 9000 psi in the steel or
8400=13.4τ1
6000 psi in the aluminum? Assume Gst = 12
τ1=626.87psi → bolts in the outer × 106 psi and Gal = 4 × 106 psi.
T=P1R1n1+P2R2n2 circle answer

T=1500π(200)(8)+1125π(150)(6) τ2=0.75(626.87)=470.15psi → bolts in the Solution 331


inner circle answer
T=3412,500πN⋅mm T=(PRn)st+(PRn)al
T=3.4125πkN⋅m=10.72kN⋅m answer T=(AτRn)st+(AτRn)al

T=1/4π(1/2)2τst(6)(6)+1/4π(3/4)2τal(4)(4)
62

T=2.25πτst+2.25πτal stress τ at the center of any rivet. Let J = developed.


ΣAρ2, where A is the area of a rivet at the
T=2.25π(τst+τal) → Equation (1)
radial distance ρ from the centroid of the
rivet group.
(τ/GR)st=(τ/GR)al

Τst/(12×106)(6)=τal/(4×106)(4) Solution 332

τst=9/2τal → Equation (2a) The shearing stress on each rivet is P/A


τ=Tρ/J
τal=2/9τat → Equation (2b)

Where:
Equations (1) and (2a) T=PRn
T=2.25π(9/2τal+τal)=12.375πτal Solution 333
ρ=R
T=12.375π(6000)=74250πlb⋅in τ=Tρ/J
2 2
J=ΣAρ =AR n
T=233.26kip⋅in
Where:
T=14(1000)(240)=3360000N⋅mm

τ=PRn(R)/AR2n J=ΣAρ2=1/4π(20)2[2(402)+2(1202)]
Equations (1) and (2b)
T=2.25π(τst+2/9τst)=2.75πτst τ=P/A (okay!) J=3200000πmm4

T=2.25π(9000)=24750πlb⋅in
This shows that τ = Tρ/J can be used to find
T=77.75kip⋅in the shearing stress at the center of any
Use T = 77.75 kip·in answer rivet.

Problem 333
A plate is fastened to a fixed member by
Problem 332 four 20-mm-diameter rivets arranged as
In a rivet group subjected to a twisting shown in Fig. P-333. Compute the
couple T, show that the torsion formula τ = maximum and minimum shearing stress
Tρ/J can be used to find the shearing
63

Maximum shearing stress (ρ = 120 mm): Solution 334 Note:


τmax=3360000(120)/3200000π The τmaximum is carried by the corner rivets (4
Without the loads P:
rivets in all) while the τminimum is carried by
τmax=40.11 MPa answer τ=Tρ/J
the middle two rivets.
Where:
Minimum shearing stress (ρ = 40 mm): T=14(10)=140kip⋅in
With the loads P, two cases will arise:
τmin=3360000(40)/3200000π
ρ=√13 in
1st case (P )
τmin=13.37 MPa answer 2 2 2 2
J=ΣAρ =1/4π(7/8) [4(√13) +2(2) ]=36.08in4 T=10(14)−6P=(140−6P)kip⋅in

τ=Tρ/J

ShowAnother way to solve for J 8000=[(140−6P)(1000)( √13)]/36.08


Problem 334
80.05=140−6P
Six 7/8-in-diameter rivets fasten the plate
in Fig. P-334 to the fixed member. Using the P=10.0 kips answer
results of Prob. 332, determine the average
shearing stress caused in each rivet by the
Note:
14 kip loads. What additional loads P can be
Without the load P = 10 kips, the shear
applied before the shearing stress in any
stress at corner rivets is 14 ksi (see τmaximum),
rivet exceeds 8000 psi?
which is way above 8 ksi. Thus, this
minimum value of P is necessary to prevent
stressing the corner rivets beyond 8 ksi.

τmaximum=140√13/36.08=14.0 ksi a 2nd case (P > 14 kips)


nswer T=6P−10(14)=(6P−140)kip⋅in
τminimum=140(2)/36.08=7.76 ksi ans τ=Tρ/J
wer
8000=[(6P−140)(1000)( √13)]/36.08

80.05=6P−140
64

P=36.68kips answer Solving for location r2=√ [(120−1603)2+402]=77.746mm


of centroid of rivets:
Problem 335
AXG=Σax
The plate shown in Fig. P-335 is fastened to
the fixed member by five 10-mm-diameter Where
J=ΣAρ2=1/4π(102)(2r12+2r22+X G 2)
rivets. Compute the value of the loads P so
that the average shearing stress in any rivet J=1/4π(102)[2(96.1482)+2(77.7462)+(1603)2]
does not exceed 70 MPa. (Hint: Use the
results of Prob. 332.) J=2624973.55mm4

A=1/2(80+160)(120)=14400mm2 ShowAnother way to solve for J


a1=a2=a3=1/2(80)(120)=4800mm2

x1=x3=1/3(120)=40mm T=120P+100P=220P
x2=2/3(120)=80mm
The critical rivets are at distance r1 from
centroid:
14400XG=4800(40)+4800(80)+4800(40)
τ=Tρ/J
XG=160/3mm
τmax=Tr1J

70=220P(96.148)/2624973.55
Solution 335
P=8686.8N answer

r1=√[(1603)2+802]=96.148mm

Potrebbero piacerti anche